Double integral with logarithms [on hold]












5












$begingroup$


Faster method to calculate the exact solution of the following integral (based on the ideas of Fedor Petrov: https://mathoverflow.net/users/4312/fedor-petrov, Double integral with logarithms, URL (version: 2019-04-15): https://mathoverflow.net/q/328126):



$$Jequiv int_{0}^{1}{int_{0}^{1}{frac{ln x-ln y}{x-y}}}dxdy .$$



Since
$$fleft( x,y right)=frac{ln x-ln y}{x-y}=fleft( y,x right),$$
the surface $fleft( x,y right) $ is symmetric with respect to the bisector plane $x = y$; so,



$$frac{J}{2}=int_{0}^{1}{dxint_{0}^{x}{frac{ln x-ln y}{x-y}}}dy.$$



With the change of variable$$yequiv tx, tin left( 0, 1 right),$$
the integral
$$int_{0}^{x}{frac{ln x-ln y}{x-y}}dy,$$
is transformed into the following one that does not depend on $x$,
$$ Iequiv -int_{0}^{1}{frac{ln t}{1-t},}dt.$$



The integration on the unit square $(0, 0), (1, 0), (1, 1), (0,1)$ is reduced to the integration on the triangle $(0, 0), (1, 0), (1, 1).$



To solve the integral $I$ we will carry out the new change of variable,
$$sequiv 1-t,$$
by which $I$ is transformed into the integral that defines the dilogarithm, whose value for $s = 1$ coincides with the Riemann zeta $zeta left( 2 right)$,whose value is well known:



$$I=int_{1}^{0}{frac{ln left( 1-s right)}{s},}ds=text{L}{{text{i}}_{2}}left( 1 right)=zeta left( 2 right)=frac{{{pi }^{2}}}{6}.$$
Therefore, the solution to the proposed integral is
$$J=frac{{{pi }^{2}}}{3}.$$



Note. I've tried it by polylogarithmic transformations, but I couldn't get the result $frac{{{pi }^{2}}}{3}.$










share|cite|improve this question









New contributor




Jesús Álvarez Lobo is a new contributor to this site. Take care in asking for clarification, commenting, and answering.
Check out our Code of Conduct.







$endgroup$



put on hold as off-topic by YCor, user44191, Pace Nielsen, Gerald Edgar, Piotr Hajlasz yesterday


This question appears to be off-topic. The users who voted to close gave this specific reason:


  • "MathOverflow is for mathematicians to ask each other questions about their research. See Math.StackExchange to ask general questions in mathematics." – Pace Nielsen, Gerald Edgar, Piotr Hajlasz

If this question can be reworded to fit the rules in the help center, please edit the question.












  • 5




    $begingroup$
    MSE is a right place for such type questions. Both Maple and Mathematica confirm $ frac{pi ^2}{3}$.
    $endgroup$
    – user64494
    yesterday






  • 9




    $begingroup$
    I do not think this is such a bad question. I would not expect even every research mathematician to come up with the answer right away. So I vote not to close.
    $endgroup$
    – RP_
    yesterday






  • 6




    $begingroup$
    @user64494 what is a general strategy: when you have an integral which you do not know how to calculate, how should you decide between MSE and MO?
    $endgroup$
    – Fedor Petrov
    yesterday






  • 6




    $begingroup$
    @user64494 being the author of several papers devoted to exact closed-form integration, I feel so old now.
    $endgroup$
    – Fedor Petrov
    yesterday






  • 2




    $begingroup$
    Learning techniques for a problem here could be illuminating (depending on ...) for future purposes of integration or summation that they relate to or approximated by (if you are into numerical values). Plus, cute arguments are aesthetically pleasing ...
    $endgroup$
    – T. Amdeberhan
    yesterday


















5












$begingroup$


Faster method to calculate the exact solution of the following integral (based on the ideas of Fedor Petrov: https://mathoverflow.net/users/4312/fedor-petrov, Double integral with logarithms, URL (version: 2019-04-15): https://mathoverflow.net/q/328126):



$$Jequiv int_{0}^{1}{int_{0}^{1}{frac{ln x-ln y}{x-y}}}dxdy .$$



Since
$$fleft( x,y right)=frac{ln x-ln y}{x-y}=fleft( y,x right),$$
the surface $fleft( x,y right) $ is symmetric with respect to the bisector plane $x = y$; so,



$$frac{J}{2}=int_{0}^{1}{dxint_{0}^{x}{frac{ln x-ln y}{x-y}}}dy.$$



With the change of variable$$yequiv tx, tin left( 0, 1 right),$$
the integral
$$int_{0}^{x}{frac{ln x-ln y}{x-y}}dy,$$
is transformed into the following one that does not depend on $x$,
$$ Iequiv -int_{0}^{1}{frac{ln t}{1-t},}dt.$$



The integration on the unit square $(0, 0), (1, 0), (1, 1), (0,1)$ is reduced to the integration on the triangle $(0, 0), (1, 0), (1, 1).$



To solve the integral $I$ we will carry out the new change of variable,
$$sequiv 1-t,$$
by which $I$ is transformed into the integral that defines the dilogarithm, whose value for $s = 1$ coincides with the Riemann zeta $zeta left( 2 right)$,whose value is well known:



$$I=int_{1}^{0}{frac{ln left( 1-s right)}{s},}ds=text{L}{{text{i}}_{2}}left( 1 right)=zeta left( 2 right)=frac{{{pi }^{2}}}{6}.$$
Therefore, the solution to the proposed integral is
$$J=frac{{{pi }^{2}}}{3}.$$



Note. I've tried it by polylogarithmic transformations, but I couldn't get the result $frac{{{pi }^{2}}}{3}.$










share|cite|improve this question









New contributor




Jesús Álvarez Lobo is a new contributor to this site. Take care in asking for clarification, commenting, and answering.
Check out our Code of Conduct.







$endgroup$



put on hold as off-topic by YCor, user44191, Pace Nielsen, Gerald Edgar, Piotr Hajlasz yesterday


This question appears to be off-topic. The users who voted to close gave this specific reason:


  • "MathOverflow is for mathematicians to ask each other questions about their research. See Math.StackExchange to ask general questions in mathematics." – Pace Nielsen, Gerald Edgar, Piotr Hajlasz

If this question can be reworded to fit the rules in the help center, please edit the question.












  • 5




    $begingroup$
    MSE is a right place for such type questions. Both Maple and Mathematica confirm $ frac{pi ^2}{3}$.
    $endgroup$
    – user64494
    yesterday






  • 9




    $begingroup$
    I do not think this is such a bad question. I would not expect even every research mathematician to come up with the answer right away. So I vote not to close.
    $endgroup$
    – RP_
    yesterday






  • 6




    $begingroup$
    @user64494 what is a general strategy: when you have an integral which you do not know how to calculate, how should you decide between MSE and MO?
    $endgroup$
    – Fedor Petrov
    yesterday






  • 6




    $begingroup$
    @user64494 being the author of several papers devoted to exact closed-form integration, I feel so old now.
    $endgroup$
    – Fedor Petrov
    yesterday






  • 2




    $begingroup$
    Learning techniques for a problem here could be illuminating (depending on ...) for future purposes of integration or summation that they relate to or approximated by (if you are into numerical values). Plus, cute arguments are aesthetically pleasing ...
    $endgroup$
    – T. Amdeberhan
    yesterday
















5












5








5


2



$begingroup$


Faster method to calculate the exact solution of the following integral (based on the ideas of Fedor Petrov: https://mathoverflow.net/users/4312/fedor-petrov, Double integral with logarithms, URL (version: 2019-04-15): https://mathoverflow.net/q/328126):



$$Jequiv int_{0}^{1}{int_{0}^{1}{frac{ln x-ln y}{x-y}}}dxdy .$$



Since
$$fleft( x,y right)=frac{ln x-ln y}{x-y}=fleft( y,x right),$$
the surface $fleft( x,y right) $ is symmetric with respect to the bisector plane $x = y$; so,



$$frac{J}{2}=int_{0}^{1}{dxint_{0}^{x}{frac{ln x-ln y}{x-y}}}dy.$$



With the change of variable$$yequiv tx, tin left( 0, 1 right),$$
the integral
$$int_{0}^{x}{frac{ln x-ln y}{x-y}}dy,$$
is transformed into the following one that does not depend on $x$,
$$ Iequiv -int_{0}^{1}{frac{ln t}{1-t},}dt.$$



The integration on the unit square $(0, 0), (1, 0), (1, 1), (0,1)$ is reduced to the integration on the triangle $(0, 0), (1, 0), (1, 1).$



To solve the integral $I$ we will carry out the new change of variable,
$$sequiv 1-t,$$
by which $I$ is transformed into the integral that defines the dilogarithm, whose value for $s = 1$ coincides with the Riemann zeta $zeta left( 2 right)$,whose value is well known:



$$I=int_{1}^{0}{frac{ln left( 1-s right)}{s},}ds=text{L}{{text{i}}_{2}}left( 1 right)=zeta left( 2 right)=frac{{{pi }^{2}}}{6}.$$
Therefore, the solution to the proposed integral is
$$J=frac{{{pi }^{2}}}{3}.$$



Note. I've tried it by polylogarithmic transformations, but I couldn't get the result $frac{{{pi }^{2}}}{3}.$










share|cite|improve this question









New contributor




Jesús Álvarez Lobo is a new contributor to this site. Take care in asking for clarification, commenting, and answering.
Check out our Code of Conduct.







$endgroup$




Faster method to calculate the exact solution of the following integral (based on the ideas of Fedor Petrov: https://mathoverflow.net/users/4312/fedor-petrov, Double integral with logarithms, URL (version: 2019-04-15): https://mathoverflow.net/q/328126):



$$Jequiv int_{0}^{1}{int_{0}^{1}{frac{ln x-ln y}{x-y}}}dxdy .$$



Since
$$fleft( x,y right)=frac{ln x-ln y}{x-y}=fleft( y,x right),$$
the surface $fleft( x,y right) $ is symmetric with respect to the bisector plane $x = y$; so,



$$frac{J}{2}=int_{0}^{1}{dxint_{0}^{x}{frac{ln x-ln y}{x-y}}}dy.$$



With the change of variable$$yequiv tx, tin left( 0, 1 right),$$
the integral
$$int_{0}^{x}{frac{ln x-ln y}{x-y}}dy,$$
is transformed into the following one that does not depend on $x$,
$$ Iequiv -int_{0}^{1}{frac{ln t}{1-t},}dt.$$



The integration on the unit square $(0, 0), (1, 0), (1, 1), (0,1)$ is reduced to the integration on the triangle $(0, 0), (1, 0), (1, 1).$



To solve the integral $I$ we will carry out the new change of variable,
$$sequiv 1-t,$$
by which $I$ is transformed into the integral that defines the dilogarithm, whose value for $s = 1$ coincides with the Riemann zeta $zeta left( 2 right)$,whose value is well known:



$$I=int_{1}^{0}{frac{ln left( 1-s right)}{s},}ds=text{L}{{text{i}}_{2}}left( 1 right)=zeta left( 2 right)=frac{{{pi }^{2}}}{6}.$$
Therefore, the solution to the proposed integral is
$$J=frac{{{pi }^{2}}}{3}.$$



Note. I've tried it by polylogarithmic transformations, but I couldn't get the result $frac{{{pi }^{2}}}{3}.$







integration






share|cite|improve this question









New contributor




Jesús Álvarez Lobo is a new contributor to this site. Take care in asking for clarification, commenting, and answering.
Check out our Code of Conduct.











share|cite|improve this question









New contributor




Jesús Álvarez Lobo is a new contributor to this site. Take care in asking for clarification, commenting, and answering.
Check out our Code of Conduct.









share|cite|improve this question




share|cite|improve this question








edited 9 hours ago







Jesús Álvarez Lobo













New contributor




Jesús Álvarez Lobo is a new contributor to this site. Take care in asking for clarification, commenting, and answering.
Check out our Code of Conduct.









asked yesterday









Jesús Álvarez LoboJesús Álvarez Lobo

353




353




New contributor




Jesús Álvarez Lobo is a new contributor to this site. Take care in asking for clarification, commenting, and answering.
Check out our Code of Conduct.





New contributor





Jesús Álvarez Lobo is a new contributor to this site. Take care in asking for clarification, commenting, and answering.
Check out our Code of Conduct.






Jesús Álvarez Lobo is a new contributor to this site. Take care in asking for clarification, commenting, and answering.
Check out our Code of Conduct.




put on hold as off-topic by YCor, user44191, Pace Nielsen, Gerald Edgar, Piotr Hajlasz yesterday


This question appears to be off-topic. The users who voted to close gave this specific reason:


  • "MathOverflow is for mathematicians to ask each other questions about their research. See Math.StackExchange to ask general questions in mathematics." – Pace Nielsen, Gerald Edgar, Piotr Hajlasz

If this question can be reworded to fit the rules in the help center, please edit the question.







put on hold as off-topic by YCor, user44191, Pace Nielsen, Gerald Edgar, Piotr Hajlasz yesterday


This question appears to be off-topic. The users who voted to close gave this specific reason:


  • "MathOverflow is for mathematicians to ask each other questions about their research. See Math.StackExchange to ask general questions in mathematics." – Pace Nielsen, Gerald Edgar, Piotr Hajlasz

If this question can be reworded to fit the rules in the help center, please edit the question.








  • 5




    $begingroup$
    MSE is a right place for such type questions. Both Maple and Mathematica confirm $ frac{pi ^2}{3}$.
    $endgroup$
    – user64494
    yesterday






  • 9




    $begingroup$
    I do not think this is such a bad question. I would not expect even every research mathematician to come up with the answer right away. So I vote not to close.
    $endgroup$
    – RP_
    yesterday






  • 6




    $begingroup$
    @user64494 what is a general strategy: when you have an integral which you do not know how to calculate, how should you decide between MSE and MO?
    $endgroup$
    – Fedor Petrov
    yesterday






  • 6




    $begingroup$
    @user64494 being the author of several papers devoted to exact closed-form integration, I feel so old now.
    $endgroup$
    – Fedor Petrov
    yesterday






  • 2




    $begingroup$
    Learning techniques for a problem here could be illuminating (depending on ...) for future purposes of integration or summation that they relate to or approximated by (if you are into numerical values). Plus, cute arguments are aesthetically pleasing ...
    $endgroup$
    – T. Amdeberhan
    yesterday
















  • 5




    $begingroup$
    MSE is a right place for such type questions. Both Maple and Mathematica confirm $ frac{pi ^2}{3}$.
    $endgroup$
    – user64494
    yesterday






  • 9




    $begingroup$
    I do not think this is such a bad question. I would not expect even every research mathematician to come up with the answer right away. So I vote not to close.
    $endgroup$
    – RP_
    yesterday






  • 6




    $begingroup$
    @user64494 what is a general strategy: when you have an integral which you do not know how to calculate, how should you decide between MSE and MO?
    $endgroup$
    – Fedor Petrov
    yesterday






  • 6




    $begingroup$
    @user64494 being the author of several papers devoted to exact closed-form integration, I feel so old now.
    $endgroup$
    – Fedor Petrov
    yesterday






  • 2




    $begingroup$
    Learning techniques for a problem here could be illuminating (depending on ...) for future purposes of integration or summation that they relate to or approximated by (if you are into numerical values). Plus, cute arguments are aesthetically pleasing ...
    $endgroup$
    – T. Amdeberhan
    yesterday










5




5




$begingroup$
MSE is a right place for such type questions. Both Maple and Mathematica confirm $ frac{pi ^2}{3}$.
$endgroup$
– user64494
yesterday




$begingroup$
MSE is a right place for such type questions. Both Maple and Mathematica confirm $ frac{pi ^2}{3}$.
$endgroup$
– user64494
yesterday




9




9




$begingroup$
I do not think this is such a bad question. I would not expect even every research mathematician to come up with the answer right away. So I vote not to close.
$endgroup$
– RP_
yesterday




$begingroup$
I do not think this is such a bad question. I would not expect even every research mathematician to come up with the answer right away. So I vote not to close.
$endgroup$
– RP_
yesterday




6




6




$begingroup$
@user64494 what is a general strategy: when you have an integral which you do not know how to calculate, how should you decide between MSE and MO?
$endgroup$
– Fedor Petrov
yesterday




$begingroup$
@user64494 what is a general strategy: when you have an integral which you do not know how to calculate, how should you decide between MSE and MO?
$endgroup$
– Fedor Petrov
yesterday




6




6




$begingroup$
@user64494 being the author of several papers devoted to exact closed-form integration, I feel so old now.
$endgroup$
– Fedor Petrov
yesterday




$begingroup$
@user64494 being the author of several papers devoted to exact closed-form integration, I feel so old now.
$endgroup$
– Fedor Petrov
yesterday




2




2




$begingroup$
Learning techniques for a problem here could be illuminating (depending on ...) for future purposes of integration or summation that they relate to or approximated by (if you are into numerical values). Plus, cute arguments are aesthetically pleasing ...
$endgroup$
– T. Amdeberhan
yesterday






$begingroup$
Learning techniques for a problem here could be illuminating (depending on ...) for future purposes of integration or summation that they relate to or approximated by (if you are into numerical values). Plus, cute arguments are aesthetically pleasing ...
$endgroup$
– T. Amdeberhan
yesterday












1 Answer
1






active

oldest

votes


















14












$begingroup$

By symmetry we have $J/2=int_0^1 dx int_0^x f(x, y) dy$ where $f(x, y) $ is your integrand. Integrating against $y$ for fixed $x$ we denote $y=tx$, $t$ varies from 0 to 1 and the integral against $y$ reads as $-int_0^1 frac{log t} {1-t}dt$. It does not depend on $x$ and is well known to be equal to $pi^2/6$ (you may use the geometric progression expansion $frac{1}{1 - t} =sum_{n>0 } t^{n-1}$ and integrate term-wise to get $sum 1/n^2$).






share|cite|improve this answer











$endgroup$




















    1 Answer
    1






    active

    oldest

    votes








    1 Answer
    1






    active

    oldest

    votes









    active

    oldest

    votes






    active

    oldest

    votes









    14












    $begingroup$

    By symmetry we have $J/2=int_0^1 dx int_0^x f(x, y) dy$ where $f(x, y) $ is your integrand. Integrating against $y$ for fixed $x$ we denote $y=tx$, $t$ varies from 0 to 1 and the integral against $y$ reads as $-int_0^1 frac{log t} {1-t}dt$. It does not depend on $x$ and is well known to be equal to $pi^2/6$ (you may use the geometric progression expansion $frac{1}{1 - t} =sum_{n>0 } t^{n-1}$ and integrate term-wise to get $sum 1/n^2$).






    share|cite|improve this answer











    $endgroup$


















      14












      $begingroup$

      By symmetry we have $J/2=int_0^1 dx int_0^x f(x, y) dy$ where $f(x, y) $ is your integrand. Integrating against $y$ for fixed $x$ we denote $y=tx$, $t$ varies from 0 to 1 and the integral against $y$ reads as $-int_0^1 frac{log t} {1-t}dt$. It does not depend on $x$ and is well known to be equal to $pi^2/6$ (you may use the geometric progression expansion $frac{1}{1 - t} =sum_{n>0 } t^{n-1}$ and integrate term-wise to get $sum 1/n^2$).






      share|cite|improve this answer











      $endgroup$
















        14












        14








        14





        $begingroup$

        By symmetry we have $J/2=int_0^1 dx int_0^x f(x, y) dy$ where $f(x, y) $ is your integrand. Integrating against $y$ for fixed $x$ we denote $y=tx$, $t$ varies from 0 to 1 and the integral against $y$ reads as $-int_0^1 frac{log t} {1-t}dt$. It does not depend on $x$ and is well known to be equal to $pi^2/6$ (you may use the geometric progression expansion $frac{1}{1 - t} =sum_{n>0 } t^{n-1}$ and integrate term-wise to get $sum 1/n^2$).






        share|cite|improve this answer











        $endgroup$



        By symmetry we have $J/2=int_0^1 dx int_0^x f(x, y) dy$ where $f(x, y) $ is your integrand. Integrating against $y$ for fixed $x$ we denote $y=tx$, $t$ varies from 0 to 1 and the integral against $y$ reads as $-int_0^1 frac{log t} {1-t}dt$. It does not depend on $x$ and is well known to be equal to $pi^2/6$ (you may use the geometric progression expansion $frac{1}{1 - t} =sum_{n>0 } t^{n-1}$ and integrate term-wise to get $sum 1/n^2$).







        share|cite|improve this answer














        share|cite|improve this answer



        share|cite|improve this answer








        edited yesterday









        T. Amdeberhan

        18.4k230132




        18.4k230132










        answered yesterday









        Fedor PetrovFedor Petrov

        52.3k6122240




        52.3k6122240















            Popular posts from this blog

            How did Captain America manage to do this?

            迪纳利

            南乌拉尔铁路局